Question

If Y = A Kα L(1-α), where α = 0.5, Y = $12,000b, K = $40,000b...

If Y = A Kα L(1-α), where α = 0.5, Y = $12,000b, K = $40,000b & L = 200 billion hrs. Find MPK & MPL, calculate by how much real GDP increases if capital stock increases by 1b and labor supply increases by one hour.

0 0
Add a comment Improve this question Transcribed image text
Know the answer?
Add Answer to:
If Y = A Kα L(1-α), where α = 0.5, Y = $12,000b, K = $40,000b...
Your Answer:

Post as a guest

Your Name:

What's your source?

Earn Coins

Coins can be redeemed for fabulous gifts.

Not the answer you're looking for? Ask your own homework help question. Our experts will answer your question WITHIN MINUTES for Free.
Similar Homework Help Questions
  • Consider the neoclassical closed economy model: Y=COY-T)+1(t) + G Y=F(K.L) M/P L(r+z* Y) CY-T) is describing...

    Consider the neoclassical closed economy model: Y=COY-T)+1(t) + G Y=F(K.L) M/P L(r+z* Y) CY-T) is describing consumptions as a function of disposable income, Kand L are fixed and do not change over time, G and T are chosen by government. And are exogenous and fixed. 1- Suppose K 150, L=500 Y-2.5 K"L- C 12+0.7(Y-T) 250 G 250, T I60-400r P 1 a 0.3 a) Calculate GDP value: I Derive the equations for marginal product of labor & marginal product of...

  • QUESTION 5 The marginal product for labor is given (MP) = 3 – 0.02*L; price of...

    QUESTION 5 The marginal product for labor is given (MP) = 3 – 0.02*L; price of the product is $100 and wage = 200.  Based on information above, the marginal product of labor at the optimal level of employment is $3 $2 $1.5 $1 2 points    QUESTION 6 If the labor elasticity of output is 0.5 and the capital elasticity of output is 0.9, then the production function exhibits constant returns to scale. economies of scale. diseconomies of scale. diminishing...

  • The negative of the slope of an isoquant on a tangent line a particular (L,K) bundle...

    The negative of the slope of an isoquant on a tangent line a particular (L,K) bundle of factor inputs is equal to the __ Assume questions are ordered top to bottom as a, b, c, d. Fig. 2: Bank Contour Plot Production Function:Q(L,K)=104 Machines 100 100 100 300 500 700 900 hr/hr (K) Human Bank Tellers hr/hr (L) 0 2 0 40 60 80 100 Oratio of the wage rate to the capital rental rate (w/r) Omarginal rate of technical...

  • Consider a firm that faces the following production function: q = f(L, K) = L1/2 K1/2...

    Consider a firm that faces the following production function: q = f(L, K) = L1/2 K1/2 where q is output, L is labor, and K is capital. Use this production function to answer the following questions. (a) What is the marginal product of labor (MPL)? (b) Does the MPL follow the law of diminishing returns? How do you know? (c) What is the marginal product of capital (MPK)? (d) Does the MPK follow the law of diminishing returns? How do...

  • 1. Suppose the production of digital cameras is characterized by the production function q F(K, L)- KL (MPL = K, MPK = L), where q represents the number of digital cameras produced. Suppose that...

    1. Suppose the production of digital cameras is characterized by the production function q F(K, L)- KL (MPL = K, MPK = L), where q represents the number of digital cameras produced. Suppose that the price of labor is $10 per unit and the price of capital is S1 per unit. (a) Graph the isoquant for q-121 000. (b) On the graph you drew for part a), draw several isocost lines including one that is tangent to the isoquant you...

  • Problem 1. Suppose that production function for the corporate sector is well presented by the Cobb-Douglas...

    Problem 1. Suppose that production function for the corporate sector is well presented by the Cobb-Douglas production function: Y = K0.3L0.7 With this production function: MPL(K, L)=0.7K0.3L0.3 and MPK(K, L)=0.3K0.7L0.7 a) Suppose the capital stock of the firm is K¯ =10. Show that the demand for labor is given by: b) Find the amount of labor demanded for W P = 0.1,W P = 0.3, W P = 0.5, W P = 0.7 and W P = 0.9. Plot the...

  • 1.The aggregate production function is Y = F(K, L) where Y = real GDP, K =...

    1.The aggregate production function is Y = F(K, L) where Y = real GDP, K = capital and L = labor. For Y = F(K, L) = K1/3Lb where b = 0 there are A.decreasing returns to scale B.increasing returns to scale C.constant returns to scale D.none of the other answers is correct. E. we cannot determine the returns to scale 2.If y = f(x) = ln U(x) then dy/dx = f’(x) = a. none of the other answers is...

  • An economy has a Cobb–Douglas production function: Y=Kα(LE)1−αY=Kα(LE)1−α The economy has a capital share of 0.30,...

    An economy has a Cobb–Douglas production function: Y=Kα(LE)1−αY=Kα(LE)1−α The economy has a capital share of 0.30, a saving rate of 42 percent, a depreciation rate of 5.00 percent, a rate of population growth of 2.50 percent, and a rate of labor-augmenting technological change of 4.0 percent. It is in steady state. . At what rates do total output and output per worker grow? Total output growth rate: % Output per worker growth rate: %

  • Question 3: Productivity, Output, and Employment (20 marks) Assume that the aggregate production is given by...

    Question 3: Productivity, Output, and Employment (20 marks) Assume that the aggregate production is given by the following: Y stands for output, K stands for the capital stock, N stands for the number of the people employed, L stands for the quantity of land used in production, and A stands for a measure of labour efficiency. a and B are parameters whose values are between O and I a) Derive an analytical expression for the marginal product of capital (MPK),...

  • Question 3: Productivity, Output, and Employment (20 marks) Assume that the aggregate production is given by...

    Question 3: Productivity, Output, and Employment (20 marks) Assume that the aggregate production is given by the following: Y stands for output, K stands for the capital stock, N stands for the number of the people employed, L stands for the quantity of land used in production, and A stands for a measure of labour efficiency. α and β are parameters whose values are between 0 and 1. a) Derive an analytical expression for the marginal product of capital (MPK),...

ADVERTISEMENT
Free Homework Help App
Download From Google Play
Scan Your Homework
to Get Instant Free Answers
Need Online Homework Help?
Ask a Question
Get Answers For Free
Most questions answered within 3 hours.
ADVERTISEMENT
ADVERTISEMENT
ADVERTISEMENT